LSAT and Law School Admissions Forum

Get expert LSAT preparation and law school admissions advice from PowerScore Test Preparation.

User avatar
 Dave Killoran
PowerScore Staff
  • PowerScore Staff
  • Posts: 5852
  • Joined: Mar 25, 2011
|
#47528
Complete Question Explanation
(The complete setup for this game can be found here: lsat/viewtopic.php?t=9394)

The correct answer choice is (E)

Answer choice (A): This answer choice violates the fourth rule.

Answer choice (B): This answer choice violates the last rule.

Answer choice (C): This answer choice violates the third rule.

Answer choice (D): This answer choice violates the second rule.

Answer choice (E): This is the correct answer choice.

Get the most out of your LSAT Prep Plus subscription.

Analyze and track your performance with our Testing and Analytics Package.